What is the kernel and range of T?












0














Define the linear transformation T by T($vec x$) = A$vec x$. Find (1) the kernel of T and (2) the range of T



$A = begin{bmatrix}1&-2&1\0&2&1end{bmatrix}$




1) Finding the Kernel(T)




$begin{bmatrix}1&-2&1\0&2&1end{bmatrix}$ => RREF $begin{bmatrix}1&0&2\0&1&.5end{bmatrix}$



$begin{bmatrix}x_1\x_2\x_3end{bmatrix}$ = $begin{bmatrix}-2x_3\-.5x_3\tend{bmatrix}$ =
$begin{bmatrix}-2t\-.5t\tend{bmatrix}$ =
t$begin{bmatrix}-2\-.5\1end{bmatrix}$



My Answer - Kernel(T) = {t(-2, -.5, -1)}, t is any real number




2) Finding the Range(T)




$begin{bmatrix}1&-2&1\0&2&1end{bmatrix}$ => RREF $begin{bmatrix}1&0&2\0&1&.5end{bmatrix}$



My Answer - Basis for range(T) = {(1.0), (0,1)}



This is what I got for the kernel and range, however the answer words it in an odd way. It says "Kernel of A is null space of A which is span{(4,1,-2)} and the Range of A is $R^2$.



Is my answer equivalent with the book's answer? Did I do anything wrong?










share|cite|improve this question


















  • 1




    The span of $(4, 1, -2)$ is the same as the span of $(-2, -1/2, 1)$, which is what you found (minus your typo in writing down the final answer from the line before it). You found a basis for the range which has dimension $2$, so the range is all of $mathbb{R}^2$.
    – T. Bongers
    Nov 30 at 0:55










  • I would be a bit pedantic and say that although saying that the basis for the range of $T$ being ${(1,0),(0,1)}$ (or their transposes if you prefer) will clearly imply that the range of $T$ is all of $Bbb R^2$, the question did ask you for what the range of $T$ was, it did not ask you what a basis for the range of $T$ was. It is like answering "The 45th president of the united states" rather than "Donald Trump" when asked the question of "Who is the current president of the united states?"
    – JMoravitz
    Nov 30 at 0:59






  • 1




    Just to check your understanding: are you saying that this is the basis for the range of $T$ because those are the first two columns of the RREF or because the RREF tells you that the rank of the matrix is 2 and so the range is all of $mathbb R^2$?
    – amd
    Nov 30 at 0:59










  • I am saying this is the basis for the range of T because those are the first two columns of the RREF that have a 1 as the first non-zero number.
    – Evan Kim
    Nov 30 at 1:03






  • 1




    That’s wrong, though in this case it happens to give you a correct answer. The pivot columns of the RREF generally do not themselves form a basis for the range (column space), but indicate which of the columns of the original matrix do. E.g., the RREF of $pmatrix{1&1\1&1}$ is $pmatrix{1&1\0&0}$, but $(1,0)^T$ is obviously not a basis for its column space.
    – amd
    Nov 30 at 22:45


















0














Define the linear transformation T by T($vec x$) = A$vec x$. Find (1) the kernel of T and (2) the range of T



$A = begin{bmatrix}1&-2&1\0&2&1end{bmatrix}$




1) Finding the Kernel(T)




$begin{bmatrix}1&-2&1\0&2&1end{bmatrix}$ => RREF $begin{bmatrix}1&0&2\0&1&.5end{bmatrix}$



$begin{bmatrix}x_1\x_2\x_3end{bmatrix}$ = $begin{bmatrix}-2x_3\-.5x_3\tend{bmatrix}$ =
$begin{bmatrix}-2t\-.5t\tend{bmatrix}$ =
t$begin{bmatrix}-2\-.5\1end{bmatrix}$



My Answer - Kernel(T) = {t(-2, -.5, -1)}, t is any real number




2) Finding the Range(T)




$begin{bmatrix}1&-2&1\0&2&1end{bmatrix}$ => RREF $begin{bmatrix}1&0&2\0&1&.5end{bmatrix}$



My Answer - Basis for range(T) = {(1.0), (0,1)}



This is what I got for the kernel and range, however the answer words it in an odd way. It says "Kernel of A is null space of A which is span{(4,1,-2)} and the Range of A is $R^2$.



Is my answer equivalent with the book's answer? Did I do anything wrong?










share|cite|improve this question


















  • 1




    The span of $(4, 1, -2)$ is the same as the span of $(-2, -1/2, 1)$, which is what you found (minus your typo in writing down the final answer from the line before it). You found a basis for the range which has dimension $2$, so the range is all of $mathbb{R}^2$.
    – T. Bongers
    Nov 30 at 0:55










  • I would be a bit pedantic and say that although saying that the basis for the range of $T$ being ${(1,0),(0,1)}$ (or their transposes if you prefer) will clearly imply that the range of $T$ is all of $Bbb R^2$, the question did ask you for what the range of $T$ was, it did not ask you what a basis for the range of $T$ was. It is like answering "The 45th president of the united states" rather than "Donald Trump" when asked the question of "Who is the current president of the united states?"
    – JMoravitz
    Nov 30 at 0:59






  • 1




    Just to check your understanding: are you saying that this is the basis for the range of $T$ because those are the first two columns of the RREF or because the RREF tells you that the rank of the matrix is 2 and so the range is all of $mathbb R^2$?
    – amd
    Nov 30 at 0:59










  • I am saying this is the basis for the range of T because those are the first two columns of the RREF that have a 1 as the first non-zero number.
    – Evan Kim
    Nov 30 at 1:03






  • 1




    That’s wrong, though in this case it happens to give you a correct answer. The pivot columns of the RREF generally do not themselves form a basis for the range (column space), but indicate which of the columns of the original matrix do. E.g., the RREF of $pmatrix{1&1\1&1}$ is $pmatrix{1&1\0&0}$, but $(1,0)^T$ is obviously not a basis for its column space.
    – amd
    Nov 30 at 22:45
















0












0








0







Define the linear transformation T by T($vec x$) = A$vec x$. Find (1) the kernel of T and (2) the range of T



$A = begin{bmatrix}1&-2&1\0&2&1end{bmatrix}$




1) Finding the Kernel(T)




$begin{bmatrix}1&-2&1\0&2&1end{bmatrix}$ => RREF $begin{bmatrix}1&0&2\0&1&.5end{bmatrix}$



$begin{bmatrix}x_1\x_2\x_3end{bmatrix}$ = $begin{bmatrix}-2x_3\-.5x_3\tend{bmatrix}$ =
$begin{bmatrix}-2t\-.5t\tend{bmatrix}$ =
t$begin{bmatrix}-2\-.5\1end{bmatrix}$



My Answer - Kernel(T) = {t(-2, -.5, -1)}, t is any real number




2) Finding the Range(T)




$begin{bmatrix}1&-2&1\0&2&1end{bmatrix}$ => RREF $begin{bmatrix}1&0&2\0&1&.5end{bmatrix}$



My Answer - Basis for range(T) = {(1.0), (0,1)}



This is what I got for the kernel and range, however the answer words it in an odd way. It says "Kernel of A is null space of A which is span{(4,1,-2)} and the Range of A is $R^2$.



Is my answer equivalent with the book's answer? Did I do anything wrong?










share|cite|improve this question













Define the linear transformation T by T($vec x$) = A$vec x$. Find (1) the kernel of T and (2) the range of T



$A = begin{bmatrix}1&-2&1\0&2&1end{bmatrix}$




1) Finding the Kernel(T)




$begin{bmatrix}1&-2&1\0&2&1end{bmatrix}$ => RREF $begin{bmatrix}1&0&2\0&1&.5end{bmatrix}$



$begin{bmatrix}x_1\x_2\x_3end{bmatrix}$ = $begin{bmatrix}-2x_3\-.5x_3\tend{bmatrix}$ =
$begin{bmatrix}-2t\-.5t\tend{bmatrix}$ =
t$begin{bmatrix}-2\-.5\1end{bmatrix}$



My Answer - Kernel(T) = {t(-2, -.5, -1)}, t is any real number




2) Finding the Range(T)




$begin{bmatrix}1&-2&1\0&2&1end{bmatrix}$ => RREF $begin{bmatrix}1&0&2\0&1&.5end{bmatrix}$



My Answer - Basis for range(T) = {(1.0), (0,1)}



This is what I got for the kernel and range, however the answer words it in an odd way. It says "Kernel of A is null space of A which is span{(4,1,-2)} and the Range of A is $R^2$.



Is my answer equivalent with the book's answer? Did I do anything wrong?







linear-algebra






share|cite|improve this question













share|cite|improve this question











share|cite|improve this question




share|cite|improve this question










asked Nov 30 at 0:49









Evan Kim

828




828








  • 1




    The span of $(4, 1, -2)$ is the same as the span of $(-2, -1/2, 1)$, which is what you found (minus your typo in writing down the final answer from the line before it). You found a basis for the range which has dimension $2$, so the range is all of $mathbb{R}^2$.
    – T. Bongers
    Nov 30 at 0:55










  • I would be a bit pedantic and say that although saying that the basis for the range of $T$ being ${(1,0),(0,1)}$ (or their transposes if you prefer) will clearly imply that the range of $T$ is all of $Bbb R^2$, the question did ask you for what the range of $T$ was, it did not ask you what a basis for the range of $T$ was. It is like answering "The 45th president of the united states" rather than "Donald Trump" when asked the question of "Who is the current president of the united states?"
    – JMoravitz
    Nov 30 at 0:59






  • 1




    Just to check your understanding: are you saying that this is the basis for the range of $T$ because those are the first two columns of the RREF or because the RREF tells you that the rank of the matrix is 2 and so the range is all of $mathbb R^2$?
    – amd
    Nov 30 at 0:59










  • I am saying this is the basis for the range of T because those are the first two columns of the RREF that have a 1 as the first non-zero number.
    – Evan Kim
    Nov 30 at 1:03






  • 1




    That’s wrong, though in this case it happens to give you a correct answer. The pivot columns of the RREF generally do not themselves form a basis for the range (column space), but indicate which of the columns of the original matrix do. E.g., the RREF of $pmatrix{1&1\1&1}$ is $pmatrix{1&1\0&0}$, but $(1,0)^T$ is obviously not a basis for its column space.
    – amd
    Nov 30 at 22:45
















  • 1




    The span of $(4, 1, -2)$ is the same as the span of $(-2, -1/2, 1)$, which is what you found (minus your typo in writing down the final answer from the line before it). You found a basis for the range which has dimension $2$, so the range is all of $mathbb{R}^2$.
    – T. Bongers
    Nov 30 at 0:55










  • I would be a bit pedantic and say that although saying that the basis for the range of $T$ being ${(1,0),(0,1)}$ (or their transposes if you prefer) will clearly imply that the range of $T$ is all of $Bbb R^2$, the question did ask you for what the range of $T$ was, it did not ask you what a basis for the range of $T$ was. It is like answering "The 45th president of the united states" rather than "Donald Trump" when asked the question of "Who is the current president of the united states?"
    – JMoravitz
    Nov 30 at 0:59






  • 1




    Just to check your understanding: are you saying that this is the basis for the range of $T$ because those are the first two columns of the RREF or because the RREF tells you that the rank of the matrix is 2 and so the range is all of $mathbb R^2$?
    – amd
    Nov 30 at 0:59










  • I am saying this is the basis for the range of T because those are the first two columns of the RREF that have a 1 as the first non-zero number.
    – Evan Kim
    Nov 30 at 1:03






  • 1




    That’s wrong, though in this case it happens to give you a correct answer. The pivot columns of the RREF generally do not themselves form a basis for the range (column space), but indicate which of the columns of the original matrix do. E.g., the RREF of $pmatrix{1&1\1&1}$ is $pmatrix{1&1\0&0}$, but $(1,0)^T$ is obviously not a basis for its column space.
    – amd
    Nov 30 at 22:45










1




1




The span of $(4, 1, -2)$ is the same as the span of $(-2, -1/2, 1)$, which is what you found (minus your typo in writing down the final answer from the line before it). You found a basis for the range which has dimension $2$, so the range is all of $mathbb{R}^2$.
– T. Bongers
Nov 30 at 0:55




The span of $(4, 1, -2)$ is the same as the span of $(-2, -1/2, 1)$, which is what you found (minus your typo in writing down the final answer from the line before it). You found a basis for the range which has dimension $2$, so the range is all of $mathbb{R}^2$.
– T. Bongers
Nov 30 at 0:55












I would be a bit pedantic and say that although saying that the basis for the range of $T$ being ${(1,0),(0,1)}$ (or their transposes if you prefer) will clearly imply that the range of $T$ is all of $Bbb R^2$, the question did ask you for what the range of $T$ was, it did not ask you what a basis for the range of $T$ was. It is like answering "The 45th president of the united states" rather than "Donald Trump" when asked the question of "Who is the current president of the united states?"
– JMoravitz
Nov 30 at 0:59




I would be a bit pedantic and say that although saying that the basis for the range of $T$ being ${(1,0),(0,1)}$ (or their transposes if you prefer) will clearly imply that the range of $T$ is all of $Bbb R^2$, the question did ask you for what the range of $T$ was, it did not ask you what a basis for the range of $T$ was. It is like answering "The 45th president of the united states" rather than "Donald Trump" when asked the question of "Who is the current president of the united states?"
– JMoravitz
Nov 30 at 0:59




1




1




Just to check your understanding: are you saying that this is the basis for the range of $T$ because those are the first two columns of the RREF or because the RREF tells you that the rank of the matrix is 2 and so the range is all of $mathbb R^2$?
– amd
Nov 30 at 0:59




Just to check your understanding: are you saying that this is the basis for the range of $T$ because those are the first two columns of the RREF or because the RREF tells you that the rank of the matrix is 2 and so the range is all of $mathbb R^2$?
– amd
Nov 30 at 0:59












I am saying this is the basis for the range of T because those are the first two columns of the RREF that have a 1 as the first non-zero number.
– Evan Kim
Nov 30 at 1:03




I am saying this is the basis for the range of T because those are the first two columns of the RREF that have a 1 as the first non-zero number.
– Evan Kim
Nov 30 at 1:03




1




1




That’s wrong, though in this case it happens to give you a correct answer. The pivot columns of the RREF generally do not themselves form a basis for the range (column space), but indicate which of the columns of the original matrix do. E.g., the RREF of $pmatrix{1&1\1&1}$ is $pmatrix{1&1\0&0}$, but $(1,0)^T$ is obviously not a basis for its column space.
– amd
Nov 30 at 22:45






That’s wrong, though in this case it happens to give you a correct answer. The pivot columns of the RREF generally do not themselves form a basis for the range (column space), but indicate which of the columns of the original matrix do. E.g., the RREF of $pmatrix{1&1\1&1}$ is $pmatrix{1&1\0&0}$, but $(1,0)^T$ is obviously not a basis for its column space.
– amd
Nov 30 at 22:45












1 Answer
1






active

oldest

votes


















1














Your answers match those of the book; they're just written differently. Note that if you set $t = -2$, you will see that the set of vectors spanned by $begin{bmatrix}-2 \ -0.5 \ 1end{bmatrix}$ is the same as that spanned by $begin{bmatrix} 4 \ 1 \ -2end{bmatrix}$. Similarly, $left{begin{bmatrix} 1 \ 0 end{bmatrix}, begin{bmatrix} 0 \ 1end{bmatrix} right}$ is a basis for $mathbb{R}^2$.






share|cite|improve this answer





















    Your Answer





    StackExchange.ifUsing("editor", function () {
    return StackExchange.using("mathjaxEditing", function () {
    StackExchange.MarkdownEditor.creationCallbacks.add(function (editor, postfix) {
    StackExchange.mathjaxEditing.prepareWmdForMathJax(editor, postfix, [["$", "$"], ["\\(","\\)"]]);
    });
    });
    }, "mathjax-editing");

    StackExchange.ready(function() {
    var channelOptions = {
    tags: "".split(" "),
    id: "69"
    };
    initTagRenderer("".split(" "), "".split(" "), channelOptions);

    StackExchange.using("externalEditor", function() {
    // Have to fire editor after snippets, if snippets enabled
    if (StackExchange.settings.snippets.snippetsEnabled) {
    StackExchange.using("snippets", function() {
    createEditor();
    });
    }
    else {
    createEditor();
    }
    });

    function createEditor() {
    StackExchange.prepareEditor({
    heartbeatType: 'answer',
    autoActivateHeartbeat: false,
    convertImagesToLinks: true,
    noModals: true,
    showLowRepImageUploadWarning: true,
    reputationToPostImages: 10,
    bindNavPrevention: true,
    postfix: "",
    imageUploader: {
    brandingHtml: "Powered by u003ca class="icon-imgur-white" href="https://imgur.com/"u003eu003c/au003e",
    contentPolicyHtml: "User contributions licensed under u003ca href="https://creativecommons.org/licenses/by-sa/3.0/"u003ecc by-sa 3.0 with attribution requiredu003c/au003e u003ca href="https://stackoverflow.com/legal/content-policy"u003e(content policy)u003c/au003e",
    allowUrls: true
    },
    noCode: true, onDemand: true,
    discardSelector: ".discard-answer"
    ,immediatelyShowMarkdownHelp:true
    });


    }
    });














    draft saved

    draft discarded


















    StackExchange.ready(
    function () {
    StackExchange.openid.initPostLogin('.new-post-login', 'https%3a%2f%2fmath.stackexchange.com%2fquestions%2f3019467%2fwhat-is-the-kernel-and-range-of-t%23new-answer', 'question_page');
    }
    );

    Post as a guest















    Required, but never shown

























    1 Answer
    1






    active

    oldest

    votes








    1 Answer
    1






    active

    oldest

    votes









    active

    oldest

    votes






    active

    oldest

    votes









    1














    Your answers match those of the book; they're just written differently. Note that if you set $t = -2$, you will see that the set of vectors spanned by $begin{bmatrix}-2 \ -0.5 \ 1end{bmatrix}$ is the same as that spanned by $begin{bmatrix} 4 \ 1 \ -2end{bmatrix}$. Similarly, $left{begin{bmatrix} 1 \ 0 end{bmatrix}, begin{bmatrix} 0 \ 1end{bmatrix} right}$ is a basis for $mathbb{R}^2$.






    share|cite|improve this answer


























      1














      Your answers match those of the book; they're just written differently. Note that if you set $t = -2$, you will see that the set of vectors spanned by $begin{bmatrix}-2 \ -0.5 \ 1end{bmatrix}$ is the same as that spanned by $begin{bmatrix} 4 \ 1 \ -2end{bmatrix}$. Similarly, $left{begin{bmatrix} 1 \ 0 end{bmatrix}, begin{bmatrix} 0 \ 1end{bmatrix} right}$ is a basis for $mathbb{R}^2$.






      share|cite|improve this answer
























        1












        1








        1






        Your answers match those of the book; they're just written differently. Note that if you set $t = -2$, you will see that the set of vectors spanned by $begin{bmatrix}-2 \ -0.5 \ 1end{bmatrix}$ is the same as that spanned by $begin{bmatrix} 4 \ 1 \ -2end{bmatrix}$. Similarly, $left{begin{bmatrix} 1 \ 0 end{bmatrix}, begin{bmatrix} 0 \ 1end{bmatrix} right}$ is a basis for $mathbb{R}^2$.






        share|cite|improve this answer












        Your answers match those of the book; they're just written differently. Note that if you set $t = -2$, you will see that the set of vectors spanned by $begin{bmatrix}-2 \ -0.5 \ 1end{bmatrix}$ is the same as that spanned by $begin{bmatrix} 4 \ 1 \ -2end{bmatrix}$. Similarly, $left{begin{bmatrix} 1 \ 0 end{bmatrix}, begin{bmatrix} 0 \ 1end{bmatrix} right}$ is a basis for $mathbb{R}^2$.







        share|cite|improve this answer












        share|cite|improve this answer



        share|cite|improve this answer










        answered Nov 30 at 0:56









        platty

        3,360320




        3,360320






























            draft saved

            draft discarded




















































            Thanks for contributing an answer to Mathematics Stack Exchange!


            • Please be sure to answer the question. Provide details and share your research!

            But avoid



            • Asking for help, clarification, or responding to other answers.

            • Making statements based on opinion; back them up with references or personal experience.


            Use MathJax to format equations. MathJax reference.


            To learn more, see our tips on writing great answers.





            Some of your past answers have not been well-received, and you're in danger of being blocked from answering.


            Please pay close attention to the following guidance:


            • Please be sure to answer the question. Provide details and share your research!

            But avoid



            • Asking for help, clarification, or responding to other answers.

            • Making statements based on opinion; back them up with references or personal experience.


            To learn more, see our tips on writing great answers.




            draft saved


            draft discarded














            StackExchange.ready(
            function () {
            StackExchange.openid.initPostLogin('.new-post-login', 'https%3a%2f%2fmath.stackexchange.com%2fquestions%2f3019467%2fwhat-is-the-kernel-and-range-of-t%23new-answer', 'question_page');
            }
            );

            Post as a guest















            Required, but never shown





















































            Required, but never shown














            Required, but never shown












            Required, but never shown







            Required, but never shown

































            Required, but never shown














            Required, but never shown












            Required, but never shown







            Required, but never shown







            Popular posts from this blog

            Different font size/position of beamer's navigation symbols template's content depending on regular/plain...

            Berounka

            I want to find a topological embedding $f : X rightarrow Y$ and $g: Y rightarrow X$, yet $X$ is not...